Which statement is true?

Which Statement Is True?

Answers

Answer 1
Statement B is true

Related Questions

Please Help !!! MATHS GCSE DATA HANDLING QUESTION. thanks xx

Answers

Answer:

A) 29men

B) 91cm

C) 13cm

Step-by-step explanation:

Men with waist of 85cm = 11

A.) Men with waist of more than 85cm = (total number of men - 11) = (40 - 11) = 29men

B.) median waist :

Median = (n) / 2

Where n = number of observations

Median = (40) / 2 = 40/2 = 20th

Taking the intersection of the 20th point on the x-axis,

Median waist = 91cm

C.) Interquartile range(IQR) = (Q3 - Q1)

Q3 = 3/4(n)

Q3 = 0.75 × 40 = 30

Q3 = 97cm

Q1 = 1/4(n)

Q1 = 0.25 × 40 = 10

Q1 = 84cm

IQR = 97 - 84 = 13cm

Brass contains copper and zinc in the ratio 2:1. How much ZINC is there in 750 g of brass?

Answers

Answer:

250

Step-by-step explanation:

copper: zinc : total

2              1      2+1 =3

We have 750 brass

750/3 = 250

Multiply each by 250

copper: zinc : total

2*250     250  3*250

500        250       750

En un programa de televisión se hacen 30 preguntas. Por cada respuesta correcta se suman 8 puntos, por cada respuesta errónea se restan 5 puntos y por aquellas preguntas que no se contesten no se suman ni se restan puntos. Si un participantes obtuvo 13 puntos, ¿Cuantas respuestas erróneas pudo tener?

Answers

Answer:

7 wrong answers.

Step-by-step explanation:

We know that when you answer a question well, you earn 8 points and a bad one loses 5 points, this means that when you answer 1 good and 1 bad, there is a total of 3 points (8-5).

This means that when answering 5 good and 5 bad, in total it would be 3 * 5 points, that is to say 15 points, that is, it goes through 2 points. To subtract 2 points, you would have to answer 1 good and 2 bad, (8 - 2 * 5), which turns out to be -2 points.

In total there would be 6 questions right and 7 questions wrong, like this:

8 * 6 + 5 * 7 = 13

13 points, in 13 questions (6 correct + 7 incorrect)

The rest of the questions were not answered so that the score does not go up or down.

Which means you got 7 wrong answers.

Identify the like terms in the expression.

Answers

Answer:D

The answer is D.

Step-by-step explanation:

Like terms are only related to the variable at the end, and the variable also has to have the same exponent or it is not a like term.

Please answer correctly !!!! Will mark brainliest !!!!!!!!!!!!

Answers

Answer:

[tex]x^2+10x+24[/tex]

Step-by-step explanation:

So just multiply the sides:

[tex]x^2+6x+4x+24[/tex]

Which is:

[tex]x^2+10x+24[/tex]


Three investors own a company. One partner owns 1/4 of the company, while the second partner owns 2/5 of the company. How
much does the third partner own?

Answers

Answer:

7/20

Step-by-step explanation:

you find the least common denominator then you add them which gives you 13/20 then you subtract 20/20 minus 13/20 which gives you 7/20

have a meh day

Answer:

[tex]\frac{7}{20}[/tex]

Step-by-step explanation:

First, add the shares of the first two investors

[tex]\frac{1}{4} +\frac{2}{5} = \frac{13}{20}[/tex]

Then, you subtract that from the total shares of the company, which is 1

[tex]1 - \frac{13}{20} = \frac{7}{20}[/tex]

Can someone please help

Answers

Answer:

The numbers to input are 2, 2 , 7

Step-by-step explanation:

252 = 2² * 3² * 7

So the numbers to input are 2, 2 , 7

Jasper decided to save $100 at the end of each month for a year and deposit it in a bank account that earns an annual interest rate of 0.3%, compounded monthly. Use the formula for an annuity, F, to determine how much money will be in the account at the end of the 6th month, rounding your answer to the nearest penny.
Note: Your interest rate must be converted to a decimal

Answers

Answer: 600.38

Step-by-step explanation:

just put in the answer you lazy

pleass more math help

Answers

Can’t answer this question

Find the value of a . A.18 B.21 C.20 D.17

Answers

Answer:

a =18

Step-by-step explanation:

The two angles are vertical angles and vertical angles are equal

6a +11 = 2a+83

Subtract 2a from each side

6a-2a +11 = 2a-2a+83

4a +11 =83

Subtract 11 from each side

4a +11 -11 = 83-11

4a = 72

divide each side by 4

4a/4 = 72/4

a =18

Which angle in ADEF has the largest measure?

Answers

Answer:

F is the largest angle

Step-by-step explanation:

The largest angle is opposite the largest side.  The smallest angle is opposite the smallest side.

The largest side is 4 so the largest angle is F

8. Where will the hour hand of a clock stop if it starts:
a.
from 7 and turns through 1 right angle?
b. from 11 and turns through 3 right angles

can you plz say me the answer​

Answers

Answer:

a. 11

b. 9

Step-by-step explanation:

thats the answer

You are going to use an incline plane to lift a heavy object to the top of a shelving unit with a height of 5 ft. The base of the incline plane is 15 ft from the shelving unit. What is the length of the incline​ plane?

Answers

Answer:

15.8 ft

Step-by-step explanation:

The inclined plane, the base of and the shelving unit form the shape of a right angled triangle.

The hypotenuse is the length of the inclined plane, h.

The base of the triangle is 15 ft.

The height of the triangle is 5 ft.

To find the hypotenuse, h, we have to use Pythagoras rule:

[tex]h^2 = a^2 + b^2[/tex]

where a = height of the triangle

b = base of the triangle

Therefore:

[tex]h^2 = 5^2 + 15^2\\\\h^2 = 25 + 225 = 250\\\\h = \sqrt{250}\\ \\h = 15.8 ft[/tex]

The inclined plane is 15.8 ft long.

A man divides his 360 cattle between his son in the ratio 7: 6: 5. Find the smallest share​

Answers

360/18=20

20*7=140

20*6=120

20*5=100

Answer=140:120:100

Answer: smallest share is 100

Step-by-step explanation:

7:6:5

Add the numbers so 7+6+5=18

Divide 360 by 18 which equals 20

Multiply each ratio by 20 so...

7×20:6×20:5×20

140:120:100

The line plot shows the result of a survey asking students how many hours they spent reading last week. How many students spent 5 or more hours reading?

Answers

Answer:

15 total students

Step-by-step explanation:

Based on the line plot picture that is attached below it can be calculated that a total of 15 students spent 5 or more hours last week reading. This can be easily calculated by adding all the x's on the marks of 5 hours or more. With both the boys and girl students combined the tally was the following

5 hours : 4 students

6 hours : 1 student

7 hours : 3 students

8 hours : 3 students

9 hours : 0 students

10 hours: 2 students

11 hours : 2 students

12 hours : 0 students

adding up to 15 total students

The lateral area of a right cone which has a base diameter of 4 units and a height of 10 units is:

Answers

Answer:

≈64.08

Step-by-step explanation:

Lateral Area=πr[tex]\sqrt{h^{2} +r^{2} }[/tex]

Radius is 1/2 of the diameter so r=2

Height=10

Substitute:

: [tex]3.141(2)\sqrt{10^{2}+2^{2} } \\6.282\sqrt{100+4} \\6.282\sqrt{104} \\ 64.066.282*10.2\\ 64.076\\\\[/tex]

64.076 rounded = 64.08

Please answer correctly !!!!!!! Will mark brainliest !!!!!!!!!!

Answers

Answer:

9 years.

Step-by-step explanation:

Make x the number of years passed.

[tex]\frac{39+x}{3+x}=4[/tex]

12 + 4x = 39 + x

27 = 3x

x = 9

The price of a computer was decreased by 7% to £500. What was the price before the decrease? Give your answer to the nearest penny.

Answers

Answer:

£537.63

Step-by-step explanation:

Let the original price of the computer be x.

The price of a computer was decreased by 7% to £500. This implies that:

x - 7/100 * x = 500

x - 0.07x = 500

0.93x = 500

x = 500/0.93 = £537.63

The price before the decrease was £537.63.

How many digits will be in the quotient?
39 4,641

Answers

Answer:

the answer is 119 so three digits

Step-by-step explanation:

Answer:

3 digits

Step-by-step explanation:

4641/39=119

What is the shape of the cross-section formed when a cylinder intersects a
plane as shown in the drawing?

Answers

Answer:

circle,

Step-by-step explanation:

disc in the middle vertically in this case is equal to the bases

The intersection is called an Oval. Hence the correct option is A. An oval in mathematics is a shape just like a circle but with an elongated outline like the shape of an egg.

What is a Cross-section?

A cross-section is a surface, an area that is created or exposed by executing a straight cut across or through a shape.

Cross-sections in technical drawings are used for depicting the internal view of an object that is three-dimensional.

Learn more about Cross-Sections are:

https://brainly.com/question/10511133

John has two jobs and earns a total of $2,345 per month. What percent of his gross income does John receive from his second job where he makes $609.70 a month?

Answers

Answer:

10000000.9

Step-by-step explanation:

HELPPPPPP ITS ABOUT EQUATIONSSSS HELPPPPP Explanation needed HELPPPPP PLEASEE THIS IS THE LAST QUESTION LEFTTTTTTT

Answers

Answer:

C. the average total cost for the first month of a gym membership

Step-by-step explanation:

x=1 is for month 1,

the value of y includes 1-month fee and one off payment, so this is the average total for the first month of membership

Answer:

It is the average total cost for the first month of a gym membership

Step-by-step explanation:

y = 34.99x+49

The 49 is the cost to join the gym and the 34.99 is the monthly cost

Let x =1 which is the cost after one month

It includes the cost to join and the 1st month membership

It is the average total cost for the first month of a gym membership

(6) Work out
5 1
6 12

Answers

Answer:

=123

Step-by-step explanation:

please solve y = 3x - 1​

Answers

Step-by-step explanation:

i think question is not complete.

(3x5−2x4−5)−(2x4+x2−10) Subtract the two polynomials

Answers

Answer:

3x^5-4x^4-x^2+5

Step-by-step explanation:

(3x^5−2x^4−5)−(2x^4+x^2−10)

Distribute the minus sign

(3x^5−2x^4−5)−2x^4-x^2+10

Combine like terms

3x^5-4x^4-x^2+5

Hello!

Answer:

[tex]\boxed{ \bf 3x^5~-~4x^4~-~x^2~+~5}[/tex]

__________________________________Explanation:

(3[tex]x^{5}[/tex] - 2[tex]x^{4}[/tex] - 5) - (2[tex]x^{4}[/tex] + x² - 10)

Drop the brackets:

3[tex]x^{5}[/tex] - 2[tex]x^{4}[/tex] - 5 - 2[tex]x^{4}[/tex] - x² + 10

Combine Like Terms:

3[tex]x^{5}[/tex] - 2[tex]x^{4}[/tex] - 2[tex]x^{4}[/tex] - x² - 5 + 10

3[tex]x^{5}[/tex] - 4[tex]x^{4}[/tex] - x² + 5

what is the greatest common factor for 9m+ 27m²

Answers

Answer:

9m

Step-by-step explanation:

Both of them are divisible by 9 and 1 m

When you factor and pull out 9m, you will get

9m (1 + 3m)

the greatest common factor for 9m and 27 m is 9

The circumference of the earth is given. 


Circumference of earth: 24,901 miles


What is the diameter of earth? Round your answer to the nearest tenth. Use 3.14 for π.


Answers

Answer:

7930.3 miles = d

Step-by-step explanation:

The circumference equals

C = pi *d

24901 = 3.14 d

Divide each side by 3.14

24901 / 3.14 = d

7930.254777 = d

Rounding to the nearest tenth

7930.3 =d


Before graduating this year, a senior homeroom was given a survey. Of those surveyed, 24% felt they learned better at home. Of this group, 80% said they plan on taking an online course in college. Of the students who felt they did not learn better at home, 40% said they plan on taking an online course in college

Part A
What is the probability a person who does not plan on taking an online course felt they learned better at home?

A : 2/21
B : 24/125
C : 38/125
D : 19/31
E :None of these

Part B
What is the probability a person who does plan on taking an online course felt they did not learn better at home?

A : 2/21
B : 24/125
C : 38/125
D : 19/31
E : None of these

Answers

Answer:

(A) The correct option is (A).

(B) The correct option is (E).

Step-by-step explanation:

The events can be defined as follows:

X =  students felt they learned better at home

Y = students plan on taking an online course in college

The information provided is:

P (X) = 0.24

P (Y|X) = 0.80

P (Y|X') = 0.40

[tex]P(Y'|X)=1-P(Y|X)\\=1-0.80\\=0.20[/tex]

[tex]P(Y'|X')=1-P(Y|X')\\=1-0.40\\=0.60[/tex]

The Bayes' theorem states that the conditional probability of an event E[tex]_{i}[/tex] given that another event A has already occurred is:

[tex]P(E_{i}|A)=\frac{P(A|E_{i})P(E_{i})}{\sum {P(A|E_{i})P(E_{i})}}[/tex]

(A)

Compute the probability a person who does not plan on taking an online course felt they learned better at home as follows:

Use the Bayes' theorem.

[tex]P(X|Y')=\frac{P(Y'|X)P(X)}{P(Y'|X)P(X)+P(Y'|X')P(X')}[/tex]

              [tex]=\frac{0.20\times 0.24}{(0.20\times 0.24)+(0.60\times 0.76)}\\\\=0.09524\\\\\approx 0.095[/tex]

Thus, the probability a person who does not plan on taking an online course felt they learned better at home is 0.095 or 2/21.

(B)

Compute the probability a person who does plan on taking an online course felt they did not learn better at home as follows:

[tex]P(X'|Y')=1-P(X|Y')\\=1-0.095\\=0.905[/tex]

Thus, the probability a person who does plan on taking an online course felt they did not learn better at home is 0.905.

What the answer to this

Answers

Answer:

I think it might be a I don't know for sure though I just need some more points so I can ask a question myself sorry if this didn't help

What is the slope of the graph? slope = -1/3 slope = -3 slope = 3 slope = 1/3

Answers

Answer:

The slope is -3

Step-by-step explanation:

This is so because the line in pointing down, leading to the fact that the slope is negative, and rise over run is 3/1......

Therefore, the slope of the line is -3

To find the slope of the line, first start with a point on the graph.

Let's use the point (0,3) which we call point A.

The other point we will call point B.

Now remember that slope can be found using the ratio rise/run

between any two points that are on that line.

To get from point A to point B along this line, we must

first go down 3 units so we say that our rise is -3.

From there we move 1 unit to the right so our run is 1.

So our slope or rise/run is -3/1 which reduces to -3.

Other Questions
What threat was the basis for the Cuban Missile Crisis?A. Soviet missilesB. economic lossesC. communist spiesD. Cuban attack What is the volume of a cone with radius 3.4 feet and altitude 4.8 feet? 4) Un pndulo tiene un perodo de oscilacin de 4 seg (T) , realizando sumovimiento desde su posicin de equilibrio, si la aplitud del movimiento es de10 cm y el ngulo de desfase es de 20 grados. Calcular:La posicin en la que se encuentra a los 5 segundos de haber iniciado elmovimiento. Brent is designing a poster that has an area of 1 sq.ft. He is going to paste a photo collage on a section of the poster that is 13 foot wide and 35 foot long. What part of a square foot will the photo collage cover? Trade routes, such as the Silk Road, contributed to the spread of religion.TrueO Falseh In desert plants, rate of water loss gets reduced due to the presence of After Halloween, a variety store had some costumes regularly priced at $20.50 on sale for 20% off the regular price. A few weeks later the costumes that hadnt sold were reduced an additional 60% off the sale price. What was the final selling price of the remaining costumes? The final selling price of costumes was $ 99 POINTS AND BRAINLIEST! PLEASE EXPLAIN. Raphael saw a square patio that was 12-feet long on each side. He wants to build a patio that will be 15-feet long on each side. The change in the scale factor is: A) 1/15 B) 3/5 C) 4/5 D) 5/4 The change of scale means that 1 inch represented 4 feet, but now 1 inch represents feet: A) 5 B) 10 C) 12 D) 15 please answer !!!!!! Zenith Consulting Co. has the following accounts in its ledger: Cash; Accounts Receivable; Supplies; Office Equipment; Accounts Payable; Common Stock; Retained Earnings; Dividends; Fees Earned; Rent Expense; Advertising Expense; Utilities Expense; Miscellaneous Expense. Transactions Mar. 1 Paid rent for the month, $2,100. 3 Paid advertising expense, $650. 5 Paid cash for supplies, $1,350. 6 Purchased office equipment on account, $9,300. 10 Received cash from customers on account, $15,600. 15 Paid creditor on account, $3,430. 27 Paid cash for miscellaneous expenses, $500. 30 Paid telephone bill for the month, $300. 31 Fees earned and billed to customers for the month, $51,230. 31 Paid electricity bill for the month, $840. 31 Paid dividends, $1,650.Journalize the preceding selected transactions for March 2018 in a two-column journal. Refer to the Chart of Accounts for exact wording of account titles.CHART OF ACCOUNTSZenith Consulting Co.General LedgerASSETS11 Cash12 Accounts Receivable13 Supplies14 Office EquipmentLIABILITIES21 Accounts PayableEQUITY31 Common Stock32 Retained Earnings33 Dividends REVENUE41 Fees EarnedEXPENSES51 Rent Expense52 Advertising Expense53 Utilities Expense54 Miscellaneous Expense Which number is equal to 7/10 What are 3 solutions for air pollution and how does it stop pollution? This problem models pollution effects in the Great Lakes. We assume pollutants are flowing into a lake at a constant rate of I kg/year, and that water is flowing out at a constant rate of F km3/year. We also assume that the pollutants are uniformly distributed throughout the lake. If C(t) denotes the concentration (in kg/km3) of pollutants at time t (in years), then C(t) satisfies the differential equationdC dt = FVC + IVwhere V is the volume of the lake (in km3). We assume that (pollutant-free) rain and streams flowing into the lake keep the volume of water in the lake constant.A) Suppose that the concentration at time t = 0 is C0. Determine the concentration at any time t by solving the differential equation.B) Find lim t[infinity] C(t) =C) For Lake Erie, V = 458 km3 and F = 175 km3/year. Suppose that one day its pollutant concentration is C0 and that all incoming pollution suddenly stopped (so I = 0). Determine the number of years it would then take for pollution levels to drop to C0/10.D) For Lake Superior, V = 12221 km3 and F = 65.2 km3/year. Abigail can read 180 words in one minute if she were to read 30 minutes each day how many days would it take to read 32,400 words of a book The first European power to establish a trading post in India was __________. A checkout clerk at a department store is expected to complete 16 transactions every hour. In the past 20 minutes, he completed 6 transactions. Choose True or False for each statement. Why is there more nitrogen than oxygen in the air? determine the range of the function graphed above Which is the most likely plot setup of a Gothic novel? Why does Sojourner Truth repeatedly ask,"And ain't I a woman?" in the secondparagraph of this speech?To argue for her inclusion in women's rights conventionsTo draw attention to her many accomplishments as areason for earning the same rights as a manTo point out that though she is a woman, she is neitherfragile nor protected from harsh realities of life, and thusdeserves the same rights as men.To make her case for the abolition of slavery after all that she has struggled against in her life.